SlideShare une entreprise Scribd logo
1  sur  68
GENERAL PRINCIPLES OF
PHARMACOLOGY
Approach to Learning
Pharmacology
Marc Imhotep Cray, M.D.
Marc Imhotep Cray, M.D.
Pedagogical Approach
2
 This course sets out to provide an understanding of scientific and
clinical pharmacology within the framework of biochemistry,
physiology and pathophysiology, travelling from drug effects on
molecular targets to their effects on the whole-organism.
In other words,
 You will be learning pharmacology using an approach that
integrates the actions of medications (drugs) starting from the
level of molecular targets (biologic effects) to the level of the
clinical patient (therapeutic and adverse effects), relying on
kindred basic medical sciences as the scaffolding.
Marc Imhotep Cray, M.D.
“What Is Pharmacology?”
…“Medical pharmacology is a bridge between basic medical
science and clinical medicine. It makes use of all the disciplines that
comprise the scientific foundation of clinical medicine; including
anatomy, physiology, pathophysiology, pathology and immunology
biochemistry, molecular and cell biology, epidemiology, genetics and
genomics. Hence…it is particularly useful for pre-clinical student to
view and engage the subject as a major horizontal and vertical
integrator, as it pulls together all the different strands of the basic
medical science years and simultaneously introduces one to the
cornerstone of modern clinical therapeutics, i.e. drugs”…
Cray MI. Integrated Scientific and Clinical Pharmacology: A Course Syllabus and
Digital Guidebook for Medical Students. Atlanta, Georgia: IVMS, 2015; 4.
3
4
Psychology
Clinical Medicine &
Therapeutics
Veterinary
Medicine
Pharmacy Biotechnology Pathology Chemistry
Psycho-
pharmacology
Clinical
Pharmacology
Veterinary
Pharmacology
Pharmaceutical
Science
Bio-
pharmaceutics
Toxicology
Medicinal
Chemistry
PHARMACOLOGY
Pharmacokinetics &
Pharmacodynamics
Biochemical
pharmacology
Molecular
pharmacology
Chemotherapy
Systems pharmacology
Neuropharmacology
Respiratory
pharmacology
Gastrointestinal
pharmacology
Immunopharmacology
Cardiovascular
pharmacology
Endocrine
pharmacology
Pharmacogenetics Pharmacogenomics Pharmacoepidemiology Pharmacoeconomics
Genetics Genomics Clinical Epidemiology Health Economics
The Scope of Pharmacology
Marc Imhotep Cray, M.D.
Importance of the GPs of Pharmacology
5
 The general principles of pharmacology are absolutely key to
gaining an understanding of how drugs exert their therapeutic and
adverse effects
 General principles are at the core of organ-systems
pharmacology, as you will be applying these principles during the
study of drug classes/organ-systems pharmacology
 Therefore, a strong knowledge of general principles will help you most
with your study of organ-systems pharmacology, as well as the prudent
application of pharmacotherapeutics during your clinical medicine training
Marc Imhotep Cray, M.D.
Learning Objectives
6
1. To understand some key terms and concepts related to the
general principles of pharmacology (pharmacokinetics and
pharmacodynamics).
2. To understand the approach to the study of disease within a
medical pharmacology and therapeutics framework.
3. To understand how one should approach the study of
pharmacology and the rational underlying. (Main Objective)
Note: The terms, concepts and mechanisms provided herein are not
intended as complete discussions. Rather, they are only introduced here as
“stair-steps” and to encapsulate Objective 3. They will be explored in more
detail further in the course of study.
Marc Imhotep Cray, M.D.
Lecture Outline
7
 Approach to Learning Pharmacology
o Key Terms and Concepts
o Mechanisms of Drug Action
• 4 Levels of complexity
 Approach to Disease
 Approach to Reading & Studying Pharmacology
o “The seven key questions”
 Key Points Capsule
 Comprehension Q & A
 Further Study: IVMS Tools and Resources
Marc Imhotep Cray, M.D.
1. Approach to Learning Pharmacology
8
Pharmacology is best learned by a systematic approach
 understanding physiology of body
 recognizing every medication (drug) has desirable and
undesirable effects and
 being aware biochemical and pharmacologic properties of a
drug affects its characteristics such as…
o duration of action
o volume of distribution
o passage through blood-brain barrier
o mechanism of elimination, and
o route of administration
Marc Imhotep Cray, M.D.
Approach to Learning Pharmacology (2)
9
 Rather than memorizing characteristics of a medication,
one should strive to learn underlying rationale for
example
 Second-generation antihistamine agents are less lipid soluble
than first-generation antihistamines  therefore 2nd Gen.
agents do not cross blood-brain barrier (BBB) as readily thus,
2nd Gen. antihistamines are not as sedating
 b/c they both bind histamine H1 receptor, efficacy is same in
treating conditions for which they are indicated (e.g., allergic
rhinitis )
Marc Imhotep Cray, M.D.
Key Terms and Concepts
10
 Pharmacology: The study of substances that interact with living
systems through biochemical processes.
 Drug (medication): A substance used in prevention, diagnosis, or
treatment of a disease or prevention of reproduction.
 Toxicology: A branch of pharmacology that studies undesirable
effects of chemicals on living organisms.
 Food and Drug Administration (FDA): Federal agency responsible
for safety & efficacy of all drugs in U.S, as well as food & cosmetics.
 Adverse effect: Also known as side effect; all unintended actions
of a drug that result from lack of specificity of drug action.
N.B. All drugs are capable of producing adverse effects>>>toxicity
Marc Imhotep Cray, M.D.
Key Terms and Concepts(2)
11
Classically there are two major divisions of pharmacology:
pharmacodynamics and pharmacokinetics
 Pharmacodynamics (PD): The actions of a drug on a living
organism, including mechanisms of action and receptor interaction.
 How the drug affects the body
 Pharmacokinetics (PK): The actions of the living organism on the
drug, including absorption, distribution, and elimination.
 How the body affects the drug
Third emerging division
 Pharmacogenomics: study of how genomic makeup affects PD &PK
 affects drug selection and application to individual patients based on
interindividual variations in the handling of drugs based on genetics
Marc Imhotep Cray, M.D.
Pharmacogenomics
12
“Pharmacogenomics may hold the opportunity of allowing
practitioners to integrate a molecular understanding of the
basis of disease with an individual's genomic makeup to
prescribe personalized, highly effective, and safe therapies.”
Roden DM. (2012) Ch. 5 Principles of Clinical Pharmacology. In: Longo DL, Fauci AS,
et al. Harrison's Principles of Internal Medicine,18th Ed., McGraw-Hill 2012, 33.
NB “Drug-gene testing is also called
pharmacogenomics, or pharmacogenetics.
All terms characterize the study of how your genes
affect your body's response to medications.”
Link to see animation and transcript: http://mayoresearch.mayo.edu/center-
for-individualized-medicine/drug-gene-testing.asp
Marc Imhotep Cray, M.D.
Major PD and PK Components & Parameters
13
Trevor AJ, Katzung BG, Kruidering-Hall M , Masters SB. Katzung & Trevor's Pharmacology Examination
& Board Review 10th Ed. New York: McGraw-Hill, 2013.
Marc Imhotep Cray, M.D.
Relationship between PK and PD
Pharmacokinetics Pharmacodynamics
Dose of drug
Drug concentration
in target organ
over time
Mechanism and
magnitude of
drug effect
Absorption
Distribution
Biotransformation
Excretion
Receptor binding
Signal transduction
Physiological effect
Redrawn after Brenner GM and Stevens CW. Pharmacology 4th ed. Sanders, 2014.
14
15
Key Terms and Concepts(4)
 Potency of drug (x-axis on dose-response curve): Relative amount of drug
needed to produce a given response
 determined by amt. of drug that reaches site of action and by affinity of
drug for receptor
 Efficacy (y-axis on dose-response curve): Drug effect as maximum response it is
able to produce
 determined by number of drug-receptor complexes and ability of
receptor to be activated once bound (intrinsic activity)
• EC50 [also ED50 in many text] refers to drug concentration that produces 50
percent of maximal response (Graded curve); whereas
• ED50 (Quantal curve) refers to drug dose that is pharmacologically effective in 50
percent of population
NB-There are two types of dose-response curves--graded and quantal--
each provides useful information for therapeutic decisions…see next slide
Marc Imhotep Cray, M.D.
Dose-Response (Concentration) Curves
16
Bardal KS, Waechter JE, Martin DS. Applied Pharmacology. St. Louis: Saunders, 2011.
Graded dose-response curve
Quantal dose-response curve:
A graph of the fraction of a population that shows a
specified response at progressively increasing doses
A graph of increasing response to increasing drug
concentration or dose
Marc Imhotep Cray, M.D.
Key Terms and Concepts(5)
17
 Agonist: A drug that activates its receptor upon binding
 Affinity and intrinsic activity; whereas, antagonists have affinity but no IA
Antagonist: A drug that attenuates effect of an agonist
 Can be competitive or non-competitive each of which can be reversible
or irreversible
 Pharmacologic antagonist: A drug that binds without activating its
receptor and thereby prevents activation by an agonist
 Competitive antagonist: A pharmacologic antagonist that can be
overcome by increasing conc. of agonist
 Non-competitive antagonist: binds to an allosteric (non-agonist)
site on receptor to prevent activation of receptor
Marc Imhotep Cray, M.D.
Key Terms and Concepts(6)
18
 Reversible antagonist: binds non-covalently to receptor
therefore can be “washed out”
 A pharmacologic antagonist that can be overcome by increasing agonist
conc.
 Irreversible antagonist: binds covalently to receptor and cannot
be displaced by either competing ligands or washing
 A pharmacologic antagonist that cannot be overcome by increasing
agonist conc.
 Physiologic antagonist: A drug that counters effects of another by
binding to a different receptor and causing opposing effects
19
Potential mechanisms of drug interaction with a receptor
Katzung BG, editor: Basic & Clinical Pharmacology, 12th ed.
New York: McGraw-Hill, 2012; Fig. 1–3.)
Possible effects resulting from these
interactions are diagrammed in dose-
response curves at right
 Ligand: A substance that forms a
complex with a biomolecule (receptor)
to serve a biological purpose
 Receptor: A molecule to which a ligand
(drug) binds to bring about a change in
function of biologic system
 Receptor site: Specific region of
receptor molecule to which drug binds
 Effector: Component of a system that
accomplishes biologic effect after
receptor is activated by an agonist
 often a channel or enzyme molecule
may be part of receptor molecule itself
Marc Imhotep Cray, M.D. 20
Routes of administration (RoA)
Path by which a drug, fluid or poison is taken into body
 Routes can also be classified based on where target of
action is
 Action may be topical (local)
 Enteral (system-wide effect, but delivered through GIT), or
 Parenteral (systemic action, but delivered by routes other
than GI tract)
Key Terms and Concepts(7)
21
Key Terms and Concepts(8)
 Route of administration: Drug may be delivered
 intravenously (IV) for delivery directly into bloodstream
 intramuscularly (IM), and
 subcutaneously (SC)
 Medication may be
 depot and slow release
 inhalant for rapid absorption & delivery to bronchi & lungs
 sublingual to bypass first-pass effect
 intrathecal for agents that penetrate BBB poorly
 rectal to avoid hepatic first-pass effect and for N/V, and
 topical administration when local effect is desired such as
dermatologic or ophthalmic agents
Lippincott Illustrated Reviews: Pharmacology
Sixth Ed. Wolters Kluwer, 2015.
Marc Imhotep Cray, M.D.
Key Terms and Concepts(9)
22
Absorption: Movement of a drug from administration site into blood
stream usually requiring crossing of one or more biologic membranes
Important parameters include
• lipid solubility
• ionization
• size of molecule and
• presence of a transport mechanism
Marc Imhotep Cray, M.D.
Key Terms and Concepts(10)
23
 Bioavailability: The percentage of an
ingested drug that is actually absorbed
into bloodstream
 By definition, intravascular doses have
100% bioavailability, f = 1.
Lippincott Illustrated Reviews: Pharmacology
Sixth Ed. Wolters Kluwer, 2015.
 Factors that influence bioavailability:
o First-pass hepatic metabolism
o Solubility of the drug
o Chemical instability
o Nature of the drug formulation
Marc Imhotep Cray, M.D.
Key Terms and Concepts(11)
24
 Volume of distribution (Vd): The size of “compartment” into
which a drug is distributed following absorption
 ratio of amt. of drug in body to drug conc. in plasma or
blood Units=liters
 determined by equation: Vd = Dose (mg) drug
administered/Initial plasma concentration (mg/L)
Marc Imhotep Cray, M.D.
Key Terms and Concepts(12)
25
 Elimination: Process by which a drug is removed from body,
generally by either metabolism (biotransformation) or excretion
 Elimination=biotransformation (liver…) + excretion (kidneys…)
 Elimination follows various kinetic models For example
 First-order kinetics describes most circumstances means that rate of drug
elimination depends on concentration of drug in plasma as described by
equation: Rate of elimination from body = Constant × Drug concentration
 Zero-order kinetics: It is less common (PEA) means that rate of
elimination is constant does not depend on the plasma drug
concentration
• consequence of a circumstance such as saturation of liver enzymes or
saturation of kidney transport mechanisms
Marc Imhotep Cray, M.D.
Key Terms and Concepts(13)
26
 Clearance: Ratio of rate of elimination of a drug to
concentration of drug in plasma or blood
 Units: volume/time, eg, mL/min or L/h
 Half-life: Time required for amount of drug in body or
blood to fall by 50%
 For drugs eliminated by first-order kinetics this number is a
constant regardless of concentration
 Units: time
Marc Imhotep Cray, M.D.
Mechanisms of Drug Action
Drug effects are produced by altering normal functions of cells and
tissues via one of four general mechanisms:
1. Interaction with receptors (major)
Ligand-activated ion channels
G-protein–coupled receptors
 Gαs-coupled receptors
 Gαi (Ginhibitory)-coupled receptors
 Gq (and G11)-coupled receptors
Intracellular nuclear receptors
Receptor-activated tyrosine kinases
2. Nonspecific chemical or physical
interactions (least common)
 e.g., antacids
3. Antimetabolite action
 e.g., ChemoTx agents
4. Alteration of the activity of enzymes
 increasing or decreasing
27
Important Notes:
• Drugs do not produce new function/s in body, but rather augment nml physiologic and biochemical
mechanisms
• No drug has a single action, but rather, both therapeutic & adverse actions or multiple effects
• Drug vs poison is dose related, as all drugs are poisons when introduced at a high enough dose
Marc Imhotep Cray, M.D. 28
Drugs act at four different levels:
1) Molecular: protein molecules are immediate targets
for most drugs. Action here translates into actions at
next level
2) Cellular: biochemical and other components of cells
participate in the process of transduction
3) Tissue: the function of heart, skin, lungs, etc., is then
altered
4) System: the function of the cardiovascular, nervous,
gastrointestinal system, etc., is then altered
Mechanisms of Drug Action (2)
Marc Imhotep Cray, M.D.
Mechanisms of Drug Action (3)
29
 To most clearly understand pharmacologic actions of
drugs (agonist and antagonist) it is necessary to know:
 which molecular targets are affected by the drug,
 nature of this molecular interaction,
 nature of the transduction system (the cellular
response),
 types of tissue that express the molecular target and
 mechanisms by which the tissue influences the body
system
NB. It is important to consider MOA of drugs
at each of the four levels of complexity.
Marc Imhotep Cray, M.D.
The four levels of MOA illustrated
30
Propranolol, a β adrenergic antagonist used to treat several
diseases including angina pectoris, a cardiac condition
resulting from localized ischemia (i.e. insufficient blood flow)
in heart:
 At the molecular level, propranolol is a competitive and
reversible antagonist to action of epinephrine (Epi) and
norepinephrine (NE) on cardiac β adrenoceptors
Marc Imhotep Cray, M.D.
Four levels of MOA cont.
31
 At the cellular level, propranolol prevents β adrenergic
agonism from elevating intracellular cyclic adenosine
monophosphate (cAMP), initiating protein
phosphorylation, Ca2+ mobilization and oxidative
metabolism
 At the tissue level, propranolol prevents β adrenergic
agonism from increasing contractile force of heart and
heart rate, i.e. it has negative inotropic and negative
chronotropic effects
Marc Imhotep Cray, M.D.
Four levels of MOA cont.
32
 At a system level, propranolol improves cardiovascular
function
 It reduces heart's β adrenergic responses to
sympathetic nervous system activity thereby
decreasing requirements for blood flow (O2 demand)
in heart tissue useful if blood supply is limited (e.g. in
coronary artery disease)
Marc Imhotep Cray, M.D.
2. Approach to Disease
33
Physicians approach clinical situations by
 taking a history (asking questions)
 performing a physical examination
 obtaining selective laboratory and imaging tests, and
 then formulating a diagnosis
The synthesis of history, physical examination, and
imaging or laboratory tests is called the clinical database
After reaching a diagnosis a treatment plan is initiated,
and patient is followed for a clinical response
Marc Imhotep Cray, M.D.
Approach to Disease (2)
34
 Rational understanding of disease (pathologic,
microbiologic, immunologic and (or) behavioral) and plans
for treatment (therapeutics) are best acquired by learning
about normal human processes on a basic science level
(physiology, biochemistry neuro and behavioral science)
 likewise, being aware of how disease alters normal
physiologic processes is also best understood on a basic
science level (pathology, pathogenesis & pathophysiology)
 Sn & Sx of disease
Marc Imhotep Cray, M.D.
Approach to Disease (3)
35
 Pharmacology and therapeutics require also ability to
tailor correct medication (drug) to patient’s situation and
awareness of medication’s adverse effect profile
 Sometimes, a patient has an adverse reaction to a drug as
chief complaint one must be able to identify medication as
culprit
 Again, an understanding of underlying basic science allows
for more rational analysis and drug (medication) choices
Marc Imhotep Cray, M.D.
3. Approach to Reading & Studying Pharmacology
36
 There are seven key questions that help to stimulate
application of basic science information to clinical setting
These are:
1. Which medications is most likely to achieve desired therapeutic
effect and/or is responsible for described symptoms or signs?
2. What is likely mechanism for clinical effect(s) and adverse effect(s)
of medication?
3. What is basic pharmacologic profile (e.g., absorption, elimination)
for medications in a certain class, and what are differences among the
agents within the class?
Marc Imhotep Cray, M.D.
Approach to Reading and Studying Pharm (2)
37
4. Given basic pharmacologic definitions such as therapeutic index (TI)
or certain safety factor (TD1/ED99), or median lethal dose (LD50), how
do medications compare in their safety profile?
5. Given a particular clinical situation with described unique patient
characteristics, which medication is most appropriate?
6. What is best treatment toxic effect of a medication?
7. What are drug-drug interactions to be cautious about regarding a
particular medication?
In the following slides we will discuss a bit more about each of these seven
key questions, explaining how they help to stimulate application of basic
science information to the clinical setting.
Marc Imhotep Cray, M.D.
1. Which drug is most likely responsible for
described symptoms or signs?
38
 One must be aware of various effects, both desirable (therapeutic)
and undesirable (ADR), produced by particular medications
 Knowledge of desirable therapeutic effects is essential in selecting
appropriate drug for particular clinical application
 Likewise, an awareness of its adverse effects (AE) is necessary, b/c
patients may present with a complaint caused by a drug effect
unaware that their symptoms are b/c of the prescribed drug
 only by being aware of common and dangerous effects can one arrive at
correct diagnosis
 Learners are encouraged not to merely memorize comparative
adverse effect profiles of drugs but rather to understand
underlying biochemical and physiological mechanisms
Marc Imhotep Cray, M.D.
2. What is mechanism for clinical effect(s) and
adverse effect(s) of drug?
39
As noted one should strive to learn underlying physiologic,
biochemical, and (or) cellular explanation for drug effect
 allows for rational choice of an alternative agent or reasonable choice of
an agent to alleviate symptoms or explanatory advice to pt. regarding
behavioral changes to diminish any adverse effects
o For example, if a 60-year-old woman who takes medications for
osteoporosis complains of severe “heartburn” one may be
suspicious, knowing that bisphosphonate medication alendronate can
cause esophagitis
• Instruction to patient to take medication while sitting upright and remaining
upright for at least 30 minutes would be proper course of action, b/c gravity will
assist in keeping alendronate in stomach rather than allowing regurgitation into
distal esophagus
Marc Imhotep Cray, M.D.
3. What is basic pharmacologic profile for drugs in
a certain class, and what are differences among
agents within said class?
40
 Understanding pharmacologic profile of medications allows
for rational therapeutics 
 instead of memorizing separate profiles for every medication,
grouping drugs together into classes allows for more efficient
learning and better comprehension
 Excellent starting point is to study how a prototype drug within a
drug class organized by structure or mechanism of action may be
used to treat a condition (such as hypertension) 
o Then within each category of agents, one should try to identify
important subclasses or drug differences
Marc Imhotep Cray, M.D.
Pharmacologic profile of a drug and differences among
agents within class (2)
41
 For example, anti-hypertensive agents can be categorized as
 diuretic agents
 β-adrenergic-blocking agents
 calcium-channel-blocking agents, and
 renin-angiotensin system inhibitors= ACEI & ARBs
 Within subclassification of renin-angiotensin system inhibitors,
angiotensin converting enzyme inhibitors (ACEI) can cause
adverse effect of a dry cough caused by increase in bradykinin
brought about by enzyme blockade
 instead, angiotensin-1 receptor blockers (ARBs) do not affect
bradykinin levels and so do not cause cough as often
Marc Imhotep Cray, M.D.
4. Given basic pharmacologic definitions such as
(TI) or (TD1/ED99), or (LD50), how do medications
compare in their safety profile?
42
 Therapeutic index (TI): Defined as TD50/ED50 (ratio of dose that
produces a toxic effect in half population to dose that produces
desired effect in half population)
 Certain safety factor (TD1/ED99): Defined as ratio of dose that
produces toxic effect in 1 percent of population to dose that
produces desired effect in 99 percent of population also known
as standard safety measure
 Median lethal dose (LD50): Defined as the median lethal dose, the
dose that will kill half the population
Marc Imhotep Cray, M.D.
TI or TD1/ED99, or LD50 (2)
43
 Based on these definitions, a desirable medication would
have a high therapeutic index (toxic dose is many times
that of efficacious dose), high certain safety factor, and
high median lethal dose (much higher than therapeutic
dose)
 Likewise, medications such as digoxin that have a low
therapeutic index require careful monitoring of bld
levels and vigilance for adverse effects
Marc Imhotep Cray, M.D.
5. Given a particular clinical situation with
described unique patient characteristics, which
medication is most appropriate?
44
One must weigh various advantages and disadvantages, as
well as different patient attributes
Some of those may include
 compliance with medications
 allergies to medications
 liver or renal insufficiency
 age
 coexisting medical disorders, and
 other medications
Marc Imhotep Cray, M.D.
Unique patient characteristics (2)
45
One must be able to sift through medication profile and
identify most dangerous adverse effects
For example,
 if a patient is already taking a monoamine-oxidase-
inhibiting agent (Selegiline) for depression then adding
a serotonin reuptake inhibitor [Fluoxetine (Prozac)]
would be potentially fatal, b/c serotonin syndrome may
ensue (hyperthermia, muscle rigidity, death)
46
Unique patient characteristics (3)
Patient profile
Age
Weight
Sex
Race
Allergies
Smoking history
Alcohol history
Diseases
Pregnant/lactating
Current therapy
Intelligence
Drug profile
Name (generic)
Class
Action
Pharmacokinetics
Indications
Contraindications/
precautions
Interactions
Adverse effects
Dosing regimen
Monitoring
Overdose/Antidote
Patient profile
 The patient is a unique individual,
with many distinguishing features that need
to be taken into account during prescribing
Drug profile
 The drug, likewise, is unique, with its
own distinguishing features
 Good prescribing involves tailoring drug
and dosing regimen to unique patient
o Clinical pharmacology provides basis
of this pharmacotherapeutic
principle
 Clinical pharmacology is a complex interaction betw. pt. and drug
Marc Imhotep Cray, M.D.
6. What is best treatment for toxic effect of a
medication?
47
 If complications of drug therapy are present one should
know proper treatment
 best learned by understanding drug MOA
For example, a pt. who has taken excessive opioids may
develop respiratory depression, caused by either a heroin
overdose or pain medication may be fatal
o Tx of an opioid overdose includes
• ABCs (airway, breathing, circulation) and
• administration of naloxone, which is a competitive
antagonist of opioids
Marc Imhotep Cray, M.D.
7. What are the drug-drug interactions to be
concerned with regarding a particular medication?
48
 Patients are often prescribed multiple medications, from
same practitioner or different clinicians
 Pts may not be aware of drug-drug interactions thus, a
clinician must compile pt. to maintain a current list of all
medications (Rx, OTC, and herbal) taken by patient
 Thus, one should be aware of most common and
dangerous drug-drug interactions
 again, understanding underlying mechanism allows for lifelong
learning rather than short-term rote memorization of facts that
are easily forgotten
Marc Imhotep Cray, M.D.
Drug-drug interactions (2)
49
 For example, magnesium sulfate to stop preterm labor
should not be used if patient is taking a calcium-channel
blocking agent such as nifedipine
 Magnesium sulfate acts as a competitive inhibitor of
calcium and by decreasing its intracellular availability
it slows down smooth muscle contraction such as in
uterus
 Calcium-channel blockers potentiate inhibition of
calcium influx and can lead to toxic effects such as
respiratory depression
Marc Imhotep Cray, M.D.
Key Points Capsule
50
❖ Understanding the pharmacologic mechanisms of drugs
allows for rational choices for therapy, fewer medication
errors, and rapid recognition and reversal of toxic effects
❖ The therapeutic index, certain safety factor (TD1/ED99), and
median lethal dose are various methods of describing the
potential toxicity of medications
❖ There are seven key questions to stimulate the application
of basic science information to the clinical arena
Marc Imhotep Cray, M.D.
Key Points Capsule (2)
51
Focus of study for each drug:
 Classification and class prototype/s (as applicable)
 Mechanism of action-biologic, therapeutic and adverse
 Indications (therapeutic use)
 Adverse effects (common vs dangerous)
 Drug-drug interactions, cautions and contraindications
 Pharmacokinetic properties, drug-disease interactions and
other patient-specific considerations
 Toxicities and antidotes (or) treatment
Marc Imhotep Cray, M.D.
“What Is Pharmacology?” A Capsule
…“Medical pharmacology is a bridge between basic science and
clinical medicine. It makes use of all the disciplines that comprise the
scientific foundation of clinical medicine; including anatomy,
physiology, pathophysiology, pathology and immunology
biochemistry, molecular and cell biology, epidemiology, genetics and
genomics. Hence…it is particularly useful for pre-clinical student to
view and engage the subject as a major horizontal and vertical
integrator, as it pulls together all the different strands of the basic
medical science years and simultaneously introduces one to the
cornerstone of modern clinical therapeutics, i.e. drugs”…
Cray MI. Integrated Scientific and Clinical Pharmacology: A Course Syllabus and
Digital Guidebook for Medical Students. Atlanta: IVMS, 2015; 4.
52
Comprehension Q & A
Marc Imhotep Cray, M.D.
Question
54
1. Bioavailability of an agent is maximal when the drug has
which of the following qualities?
A. Highly lipid soluble
B. More than 100 Daltons in molecular weight
C. Highly bound to plasma proteins
D. Highly ionized
Marc Imhotep Cray, M.D.
Answer
55
1. A. Transport across biologic membranes and thus
bioavailability is maximal with high lipid solubility.
Marc Imhotep Cray, M.D.
Question
56
2. An agent is noted to have a very low calculated volume
of distribution (Vd). Which of the following is the best
explanation?
A. The agent is eliminated by the kidneys, and the patient
has renal insufficiency.
B. The agent is extensively bound to plasma proteins.
C. The agent is extensively sequestered in tissue.
D. The agent is eliminated by zero-order kinetics.
Marc Imhotep Cray, M.D.
Answer
57
2. B. The volume of distribution is calculated by
administering a known dose of drug (mg) IV and then
measuring an initial plasma concentration (mg/L). The ratio
of the mass of drug given (mg) divided by the initial plasma
concentration (mg/L) gives the Vd. A very low Vd may
indicate extensive protein binding (drug is sequestered in the
bloodstream), whereas a high Vd may indicate extensive
tissue binding (drug is sequestered in the tissue).
Marc Imhotep Cray, M.D.
Question
58
3. Which of the following describes the first-pass effect?
A. Inactivation of a drug as a result of the gastric acids.
B. Absorption of a drug through the duodenum.
C. Drug given orally is metabolized by the liver before
entering the circulation.
D. Drug given IV accumulates quickly in the central nervous
system (CNS).
Marc Imhotep Cray, M.D.
Answer
59
3. C. The first-pass effect refers to the process in which
following oral administration a drug is extensively
metabolized as it initially passes through the liver, before it
enters the general circulation. Liver enzymes may metabolize
the agent to such an extent that the drug cannot be
administered orally.
Marc Imhotep Cray, M.D.
Question
60
4. A laboratory experiment is being conducted in which a mammal is
injected with a noncompetitive antagonist to the histamine receptor.
Which of the following best describes this agent?
A. The drug binds to the histamine receptor and partially activates it.
B. The drug binds to the histamine receptor but does not activate it.
C. The drug binds to the receptor, but not where histamine binds, and
prevents the receptor from being activated.
D. The drug irreversibly binds to the histamine receptor and renders it
ineffective.
Marc Imhotep Cray, M.D.
Answer
61
4. C. A noncompetitive antagonist binds to the receptor at a
site other than the agonist-binding site and renders it less
effective by preventing agonist binding or preventing
activation.
Marc Imhotep Cray, M.D.
Question
62
5. A 25-year-old medical student is given a prescription for asthma,
which the physician states has a very high therapeutic index. Which
of the statements best characterizes the drug as it relates to the
therapeutic index?
A. The drug’s serum levels will likely need to be carefully
monitored.
B. The drug is likely to cross the blood-brain barrier.
C. The drug is likely to have extensive drug-drug interactions.
D. The drug is unlikely to have any serious adverse effects.
Marc Imhotep Cray, M.D.
Answer
63
5. D. An agent with a high therapeutic index means the toxic
dose is very much higher than the therapeutic dose, and it is
less likely to produce toxic effects at therapeutic levels.
Marc Imhotep Cray, M.D.
Question
64
6. A drug M is injected IV into a laboratory subject. It is noted
to have high serum protein binding. Which of the following is
most likely to be increased as a result?
A. Drug interaction
B. Distribution of the drug to tissue sites
C. Renal excretion
D. Liver metabolism
Marc Imhotep Cray, M.D.
Answer
65
6. A. High protein binding means less drug to the tissue, the
kidney, and the liver. Drug interaction may occur if the agent
binds to the same protein site as other drugs, thus displacing
drugs and increasing serum levels.
Marc Imhotep Cray, M.D.
Question
66
7. A bolus of drug K is given IV. The drug is noted to follow
first-order kinetics. Which of the following describes the
elimination of drug K?
A. The rate of elimination of drug K is constant.
B. The rate of elimination of drug K is proportional to the
patient’s renal function.
C. The rate of elimination of drug K is proportional to its
concentration in the patient’s plasma.
D. The rate of elimination of drug K is dependent on a
nonlinear relationship to the plasma protein concentration.
Marc Imhotep Cray, M.D.
Answer
67
7. C. First-order kinetics means the rate of elimination of a
drug is proportional to the plasma concentration.
Marc Imhotep Cray, M.D.
Further study tools and resources:
68
IVMS Online Medical Pharmacology Course (5 components):
Instructor: Marc Imhotep Cray, M.D. Course Website: Link
 Integrated Scientific and Clinical Pharmacology: A MS1 & MS2 Course Syllabus and Digital
Guidebook (2015)
• Medical Pharmacology: Core Concepts and Learning Objectives
• Medical Pharmacology Case Studies
• Medical Pharmacology Unit e-Notes
• Medical Pharmacology Glossary of Terms
e-Learning resource center: IVMS Medical Pharmacology Cloud Folder

Contenu connexe

Tendances

Expt. 9 Effect of atropine on DRC of acetylcholine using rat ileum
Expt. 9  Effect of atropine on DRC of acetylcholine using rat ileumExpt. 9  Effect of atropine on DRC of acetylcholine using rat ileum
Expt. 9 Effect of atropine on DRC of acetylcholine using rat ileumVISHALJADHAV100
 
Adverse Drug Reaction
Adverse Drug ReactionAdverse Drug Reaction
Adverse Drug Reactionsunayanamali
 
Immunopharmacology
ImmunopharmacologyImmunopharmacology
ImmunopharmacologyManish Kumar
 
Historical developmental and scope of pharmacology
Historical developmental and scope of pharmacologyHistorical developmental and scope of pharmacology
Historical developmental and scope of pharmacologybalaji college of pharmacy
 
Drugs used in protozoal infections-Mr. panneh
Drugs used in protozoal infections-Mr. pannehDrugs used in protozoal infections-Mr. panneh
Drugs used in protozoal infections-Mr. pannehabdou panneh
 
Endocrine pharmacology
Endocrine pharmacologyEndocrine pharmacology
Endocrine pharmacologyPavana K A
 
Pharmacokinetics / Biopharmaceutics - Multi compartment IV bolus
Pharmacokinetics / Biopharmaceutics - Multi compartment IV bolusPharmacokinetics / Biopharmaceutics - Multi compartment IV bolus
Pharmacokinetics / Biopharmaceutics - Multi compartment IV bolusAreej Abu Hanieh
 
Expt. 8 Effect of physostigmine on DRC of acetylcholine using frog rectus abd...
Expt. 8 Effect of physostigmine on DRC of acetylcholine using frog rectus abd...Expt. 8 Effect of physostigmine on DRC of acetylcholine using frog rectus abd...
Expt. 8 Effect of physostigmine on DRC of acetylcholine using frog rectus abd...VISHALJADHAV100
 
Suhas (chronopharmacology of diabetes)
Suhas (chronopharmacology of diabetes)Suhas (chronopharmacology of diabetes)
Suhas (chronopharmacology of diabetes)suhaspatil114
 
Drug idiosyncrasy
Drug idiosyncrasyDrug idiosyncrasy
Drug idiosyncrasyNafeeyabano
 
Antidiabetic drug-1
Antidiabetic drug-1Antidiabetic drug-1
Antidiabetic drug-1NajirRuman
 
Clinical Pharmacokinetics-I [half life, order of kinetics, steady state]
Clinical Pharmacokinetics-I [half life, order of kinetics, steady state]Clinical Pharmacokinetics-I [half life, order of kinetics, steady state]
Clinical Pharmacokinetics-I [half life, order of kinetics, steady state]BADAR UDDIN UMAR
 
pharmacokinetic drug interactions
 pharmacokinetic drug interactions pharmacokinetic drug interactions
pharmacokinetic drug interactionsSyed Imran
 
Biological agents as drugs
Biological agents as drugsBiological agents as drugs
Biological agents as drugsMohammad Hussain
 
DRUG INTERACTIONS (MECHANISMS OF DRUG-DRUG INTERACTIONS)
DRUG INTERACTIONS (MECHANISMS OF DRUG-DRUG INTERACTIONS)DRUG INTERACTIONS (MECHANISMS OF DRUG-DRUG INTERACTIONS)
DRUG INTERACTIONS (MECHANISMS OF DRUG-DRUG INTERACTIONS)N Anusha
 
Estimation of pharmacokinetic parameters
Estimation of pharmacokinetic parametersEstimation of pharmacokinetic parameters
Estimation of pharmacokinetic parametersKarun Kumar
 
Autacoids - pharmacological actions and drugs related to them.
Autacoids - pharmacological actions and drugs related to them. Autacoids - pharmacological actions and drugs related to them.
Autacoids - pharmacological actions and drugs related to them. SIVASWAROOP YARASI
 
Drugs used in pregnancy and lactation
Drugs used in pregnancy and lactationDrugs used in pregnancy and lactation
Drugs used in pregnancy and lactationKoppala RVS Chaitanya
 

Tendances (20)

Expt. 9 Effect of atropine on DRC of acetylcholine using rat ileum
Expt. 9  Effect of atropine on DRC of acetylcholine using rat ileumExpt. 9  Effect of atropine on DRC of acetylcholine using rat ileum
Expt. 9 Effect of atropine on DRC of acetylcholine using rat ileum
 
Adverse Drug Reaction
Adverse Drug ReactionAdverse Drug Reaction
Adverse Drug Reaction
 
Immunopharmacology
ImmunopharmacologyImmunopharmacology
Immunopharmacology
 
Historical developmental and scope of pharmacology
Historical developmental and scope of pharmacologyHistorical developmental and scope of pharmacology
Historical developmental and scope of pharmacology
 
Drugs used in protozoal infections-Mr. panneh
Drugs used in protozoal infections-Mr. pannehDrugs used in protozoal infections-Mr. panneh
Drugs used in protozoal infections-Mr. panneh
 
Endocrine pharmacology
Endocrine pharmacologyEndocrine pharmacology
Endocrine pharmacology
 
Protein binding
Protein bindingProtein binding
Protein binding
 
Pharmacokinetics / Biopharmaceutics - Multi compartment IV bolus
Pharmacokinetics / Biopharmaceutics - Multi compartment IV bolusPharmacokinetics / Biopharmaceutics - Multi compartment IV bolus
Pharmacokinetics / Biopharmaceutics - Multi compartment IV bolus
 
Expt. 8 Effect of physostigmine on DRC of acetylcholine using frog rectus abd...
Expt. 8 Effect of physostigmine on DRC of acetylcholine using frog rectus abd...Expt. 8 Effect of physostigmine on DRC of acetylcholine using frog rectus abd...
Expt. 8 Effect of physostigmine on DRC of acetylcholine using frog rectus abd...
 
Suhas (chronopharmacology of diabetes)
Suhas (chronopharmacology of diabetes)Suhas (chronopharmacology of diabetes)
Suhas (chronopharmacology of diabetes)
 
Drug idiosyncrasy
Drug idiosyncrasyDrug idiosyncrasy
Drug idiosyncrasy
 
Antidiabetic drug-1
Antidiabetic drug-1Antidiabetic drug-1
Antidiabetic drug-1
 
Clinical Pharmacokinetics-I [half life, order of kinetics, steady state]
Clinical Pharmacokinetics-I [half life, order of kinetics, steady state]Clinical Pharmacokinetics-I [half life, order of kinetics, steady state]
Clinical Pharmacokinetics-I [half life, order of kinetics, steady state]
 
pharmacokinetic drug interactions
 pharmacokinetic drug interactions pharmacokinetic drug interactions
pharmacokinetic drug interactions
 
Biological agents as drugs
Biological agents as drugsBiological agents as drugs
Biological agents as drugs
 
DRUG INTERACTIONS (MECHANISMS OF DRUG-DRUG INTERACTIONS)
DRUG INTERACTIONS (MECHANISMS OF DRUG-DRUG INTERACTIONS)DRUG INTERACTIONS (MECHANISMS OF DRUG-DRUG INTERACTIONS)
DRUG INTERACTIONS (MECHANISMS OF DRUG-DRUG INTERACTIONS)
 
Estimation of pharmacokinetic parameters
Estimation of pharmacokinetic parametersEstimation of pharmacokinetic parameters
Estimation of pharmacokinetic parameters
 
Autacoids - pharmacological actions and drugs related to them.
Autacoids - pharmacological actions and drugs related to them. Autacoids - pharmacological actions and drugs related to them.
Autacoids - pharmacological actions and drugs related to them.
 
Drug absorption
Drug absorptionDrug absorption
Drug absorption
 
Drugs used in pregnancy and lactation
Drugs used in pregnancy and lactationDrugs used in pregnancy and lactation
Drugs used in pregnancy and lactation
 

En vedette

General Principles in Pharmacology
General Principles in PharmacologyGeneral Principles in Pharmacology
General Principles in PharmacologyDJ CrissCross
 
Introduction to basic principles of pharmacology
Introduction to basic principles of pharmacologyIntroduction to basic principles of pharmacology
Introduction to basic principles of pharmacologyBalmukund Thakkar
 
Pharmacological principles
Pharmacological principlesPharmacological principles
Pharmacological principlesraj kumar
 
Pharmacology Review Chapter 29-38
Pharmacology Review Chapter 29-38Pharmacology Review Chapter 29-38
Pharmacology Review Chapter 29-38Carrie Wyatt
 
Internal Medicine Board Review - Rheumatology Flashcards - by Knowmedge
Internal Medicine Board Review - Rheumatology Flashcards -  by KnowmedgeInternal Medicine Board Review - Rheumatology Flashcards -  by Knowmedge
Internal Medicine Board Review - Rheumatology Flashcards - by KnowmedgeKnowmedge
 
Basic concepts of pharmacology
Basic concepts of pharmacologyBasic concepts of pharmacology
Basic concepts of pharmacologydraadil
 
Adams ch02 lecture
Adams ch02 lectureAdams ch02 lecture
Adams ch02 lectureTheSlaps
 
NurseReview.Org - Antianginal Agents Updates (pharmacology classes)
NurseReview.Org - Antianginal Agents Updates (pharmacology classes)NurseReview.Org - Antianginal Agents Updates (pharmacology classes)
NurseReview.Org - Antianginal Agents Updates (pharmacology classes)jben501
 
Pharmacology
PharmacologyPharmacology
PharmacologysilGERSIL
 
Sodium and potassium
Sodium and potassiumSodium and potassium
Sodium and potassium101010101965
 
THE ROLE OF PEDIATRICIAN IN PRIMARY PREVENTION OF OBISITY
THE ROLE OF PEDIATRICIAN IN PRIMARY PREVENTION OF OBISITYTHE ROLE OF PEDIATRICIAN IN PRIMARY PREVENTION OF OBISITY
THE ROLE OF PEDIATRICIAN IN PRIMARY PREVENTION OF OBISITYmandar haval
 
Renal Insufficiency and Dialysis, Urinary Incontinence and Urinary Tract Calculi
Renal Insufficiency and Dialysis, Urinary Incontinence and Urinary Tract CalculiRenal Insufficiency and Dialysis, Urinary Incontinence and Urinary Tract Calculi
Renal Insufficiency and Dialysis, Urinary Incontinence and Urinary Tract CalculiImhotep Virtual Medical School
 
Bb1 15 past lecture summaries
Bb1 15 past lecture summariesBb1 15 past lecture summaries
Bb1 15 past lecture summariesMelody Smith
 
IVMS Comprehensive -USMLE-type- Pharmacology-#1-2012 Examination-answers and ...
IVMS Comprehensive -USMLE-type- Pharmacology-#1-2012 Examination-answers and ...IVMS Comprehensive -USMLE-type- Pharmacology-#1-2012 Examination-answers and ...
IVMS Comprehensive -USMLE-type- Pharmacology-#1-2012 Examination-answers and ...Imhotep Virtual Medical School
 
GMEC - Fluid and Electrolyte Imbalances in Emergency Nursing
GMEC - Fluid and Electrolyte Imbalances in Emergency NursingGMEC - Fluid and Electrolyte Imbalances in Emergency Nursing
GMEC - Fluid and Electrolyte Imbalances in Emergency NursingOpen.Michigan
 

En vedette (20)

General Principles in Pharmacology
General Principles in PharmacologyGeneral Principles in Pharmacology
General Principles in Pharmacology
 
Introduction to basic principles of pharmacology
Introduction to basic principles of pharmacologyIntroduction to basic principles of pharmacology
Introduction to basic principles of pharmacology
 
INTRODUCTION TO PHARMACOLOGY
INTRODUCTION TO PHARMACOLOGYINTRODUCTION TO PHARMACOLOGY
INTRODUCTION TO PHARMACOLOGY
 
Basic parmacology
Basic parmacologyBasic parmacology
Basic parmacology
 
Pharmacological principles
Pharmacological principlesPharmacological principles
Pharmacological principles
 
Pharmacology Review Chapter 29-38
Pharmacology Review Chapter 29-38Pharmacology Review Chapter 29-38
Pharmacology Review Chapter 29-38
 
Drugs Used In Disorders of Gastrointestinal System
Drugs Used In Disorders of Gastrointestinal SystemDrugs Used In Disorders of Gastrointestinal System
Drugs Used In Disorders of Gastrointestinal System
 
Internal Medicine Board Review - Rheumatology Flashcards - by Knowmedge
Internal Medicine Board Review - Rheumatology Flashcards -  by KnowmedgeInternal Medicine Board Review - Rheumatology Flashcards -  by Knowmedge
Internal Medicine Board Review - Rheumatology Flashcards - by Knowmedge
 
Basic concepts of pharmacology
Basic concepts of pharmacologyBasic concepts of pharmacology
Basic concepts of pharmacology
 
An Introduction to Pharmacology
An Introduction to PharmacologyAn Introduction to Pharmacology
An Introduction to Pharmacology
 
Adams ch02 lecture
Adams ch02 lectureAdams ch02 lecture
Adams ch02 lecture
 
NurseReview.Org - Antianginal Agents Updates (pharmacology classes)
NurseReview.Org - Antianginal Agents Updates (pharmacology classes)NurseReview.Org - Antianginal Agents Updates (pharmacology classes)
NurseReview.Org - Antianginal Agents Updates (pharmacology classes)
 
Pharmacology
PharmacologyPharmacology
Pharmacology
 
Sodium and potassium
Sodium and potassiumSodium and potassium
Sodium and potassium
 
THE ROLE OF PEDIATRICIAN IN PRIMARY PREVENTION OF OBISITY
THE ROLE OF PEDIATRICIAN IN PRIMARY PREVENTION OF OBISITYTHE ROLE OF PEDIATRICIAN IN PRIMARY PREVENTION OF OBISITY
THE ROLE OF PEDIATRICIAN IN PRIMARY PREVENTION OF OBISITY
 
Renal Insufficiency and Dialysis, Urinary Incontinence and Urinary Tract Calculi
Renal Insufficiency and Dialysis, Urinary Incontinence and Urinary Tract CalculiRenal Insufficiency and Dialysis, Urinary Incontinence and Urinary Tract Calculi
Renal Insufficiency and Dialysis, Urinary Incontinence and Urinary Tract Calculi
 
Bb1 15 past lecture summaries
Bb1 15 past lecture summariesBb1 15 past lecture summaries
Bb1 15 past lecture summaries
 
IVMS Comprehensive -USMLE-type- Pharmacology-#1-2012 Examination-answers and ...
IVMS Comprehensive -USMLE-type- Pharmacology-#1-2012 Examination-answers and ...IVMS Comprehensive -USMLE-type- Pharmacology-#1-2012 Examination-answers and ...
IVMS Comprehensive -USMLE-type- Pharmacology-#1-2012 Examination-answers and ...
 
Basic EKG Notes
Basic EKG NotesBasic EKG Notes
Basic EKG Notes
 
GMEC - Fluid and Electrolyte Imbalances in Emergency Nursing
GMEC - Fluid and Electrolyte Imbalances in Emergency NursingGMEC - Fluid and Electrolyte Imbalances in Emergency Nursing
GMEC - Fluid and Electrolyte Imbalances in Emergency Nursing
 

Similaire à General Princples of Pharmacology_Approach to Learning Pharmacology

Pharmacology Definitions, Nomenclature and Pharmacodynamics I
Pharmacology Definitions, Nomenclature and Pharmacodynamics IPharmacology Definitions, Nomenclature and Pharmacodynamics I
Pharmacology Definitions, Nomenclature and Pharmacodynamics IImhotep Virtual Medical School
 
Introduction to pharmacokinetics and pharmacodynamics 2022.doc
Introduction to pharmacokinetics and pharmacodynamics 2022.docIntroduction to pharmacokinetics and pharmacodynamics 2022.doc
Introduction to pharmacokinetics and pharmacodynamics 2022.docAhmed Ali
 
introductiontopharmacoepidemiology-230613144442-c713d639.pdf
introductiontopharmacoepidemiology-230613144442-c713d639.pdfintroductiontopharmacoepidemiology-230613144442-c713d639.pdf
introductiontopharmacoepidemiology-230613144442-c713d639.pdfOgunsina1
 
INTRODUCTION TO PHARMACOEPIDEMIOLOGY.pptx
INTRODUCTION TO PHARMACOEPIDEMIOLOGY.pptxINTRODUCTION TO PHARMACOEPIDEMIOLOGY.pptx
INTRODUCTION TO PHARMACOEPIDEMIOLOGY.pptxAmeena Kadar
 
Pharmacoepidemiology... Pharmacovigilan e
Pharmacoepidemiology... Pharmacovigilan ePharmacoepidemiology... Pharmacovigilan e
Pharmacoepidemiology... Pharmacovigilan esonalinghatmal
 
Orientation To Pharmacology
Orientation To PharmacologyOrientation To Pharmacology
Orientation To PharmacologyTpetrici
 
Pharmacology Theory_Introduction & Routes of Administration.pptx
Pharmacology Theory_Introduction & Routes of Administration.pptxPharmacology Theory_Introduction & Routes of Administration.pptx
Pharmacology Theory_Introduction & Routes of Administration.pptxAbhishekSharma921450
 
Gen Pharmacology Intro Bds
Gen Pharmacology Intro BdsGen Pharmacology Intro Bds
Gen Pharmacology Intro BdsRathnakar U P
 
Session 1 part 1
Session 1 part 1Session 1 part 1
Session 1 part 1plmiami
 
Introduction Of Pharmacology
Introduction Of PharmacologyIntroduction Of Pharmacology
Introduction Of Pharmacologyshabeel pn
 
The Evolution of Future Medicine - WE Medicine - To Meet Unmet Medical Needs_...
The Evolution of Future Medicine - WE Medicine - To Meet Unmet Medical Needs_...The Evolution of Future Medicine - WE Medicine - To Meet Unmet Medical Needs_...
The Evolution of Future Medicine - WE Medicine - To Meet Unmet Medical Needs_...CrimsonpublishersCancer
 
Chapter 1- MSC PKs & PDs -Ok11.pdf
Chapter 1- MSC PKs & PDs -Ok11.pdfChapter 1- MSC PKs & PDs -Ok11.pdf
Chapter 1- MSC PKs & PDs -Ok11.pdfmergawekwaya
 
Executive summary molecular medicine
Executive summary molecular medicineExecutive summary molecular medicine
Executive summary molecular medicineRyan Witt
 

Similaire à General Princples of Pharmacology_Approach to Learning Pharmacology (20)

Pharmacology Definitions, Nomenclature and Pharmacodynamics I
Pharmacology Definitions, Nomenclature and Pharmacodynamics IPharmacology Definitions, Nomenclature and Pharmacodynamics I
Pharmacology Definitions, Nomenclature and Pharmacodynamics I
 
Introduction to pharmacology
Introduction to pharmacologyIntroduction to pharmacology
Introduction to pharmacology
 
Pharmacology
PharmacologyPharmacology
Pharmacology
 
Explore pharmacology
Explore pharmacologyExplore pharmacology
Explore pharmacology
 
Introduction to pharmacokinetics and pharmacodynamics 2022.doc
Introduction to pharmacokinetics and pharmacodynamics 2022.docIntroduction to pharmacokinetics and pharmacodynamics 2022.doc
Introduction to pharmacokinetics and pharmacodynamics 2022.doc
 
Pgy 3220 course handbook 2014
Pgy 3220 course handbook  2014Pgy 3220 course handbook  2014
Pgy 3220 course handbook 2014
 
introductiontopharmacoepidemiology-230613144442-c713d639.pdf
introductiontopharmacoepidemiology-230613144442-c713d639.pdfintroductiontopharmacoepidemiology-230613144442-c713d639.pdf
introductiontopharmacoepidemiology-230613144442-c713d639.pdf
 
INTRODUCTION TO PHARMACOEPIDEMIOLOGY.pptx
INTRODUCTION TO PHARMACOEPIDEMIOLOGY.pptxINTRODUCTION TO PHARMACOEPIDEMIOLOGY.pptx
INTRODUCTION TO PHARMACOEPIDEMIOLOGY.pptx
 
Pharmacoepidemiology... Pharmacovigilan e
Pharmacoepidemiology... Pharmacovigilan ePharmacoepidemiology... Pharmacovigilan e
Pharmacoepidemiology... Pharmacovigilan e
 
Orientation To Pharmacology
Orientation To PharmacologyOrientation To Pharmacology
Orientation To Pharmacology
 
Pharmacology Theory_Introduction & Routes of Administration.pptx
Pharmacology Theory_Introduction & Routes of Administration.pptxPharmacology Theory_Introduction & Routes of Administration.pptx
Pharmacology Theory_Introduction & Routes of Administration.pptx
 
Gen Pharmacology Intro Bds
Gen Pharmacology Intro BdsGen Pharmacology Intro Bds
Gen Pharmacology Intro Bds
 
Session 1 part 1
Session 1 part 1Session 1 part 1
Session 1 part 1
 
A story of drug development
A story of drug developmentA story of drug development
A story of drug development
 
Introduction Of Pharmacology
Introduction Of PharmacologyIntroduction Of Pharmacology
Introduction Of Pharmacology
 
The Evolution of Future Medicine - WE Medicine - To Meet Unmet Medical Needs_...
The Evolution of Future Medicine - WE Medicine - To Meet Unmet Medical Needs_...The Evolution of Future Medicine - WE Medicine - To Meet Unmet Medical Needs_...
The Evolution of Future Medicine - WE Medicine - To Meet Unmet Medical Needs_...
 
Pharma 2016
Pharma 2016Pharma 2016
Pharma 2016
 
Chapter 1- MSC PKs & PDs -Ok11.pdf
Chapter 1- MSC PKs & PDs -Ok11.pdfChapter 1- MSC PKs & PDs -Ok11.pdf
Chapter 1- MSC PKs & PDs -Ok11.pdf
 
Executive summary molecular medicine
Executive summary molecular medicineExecutive summary molecular medicine
Executive summary molecular medicine
 
Scope of Pharmacology
Scope of Pharmacology Scope of Pharmacology
Scope of Pharmacology
 

Plus de Imhotep Virtual Medical School

Oncologic Pathology_A Case-based Organ Systems Review (USMLE Step 1)
Oncologic Pathology_A Case-based Organ Systems Review (USMLE Step 1)Oncologic Pathology_A Case-based Organ Systems Review (USMLE Step 1)
Oncologic Pathology_A Case-based Organ Systems Review (USMLE Step 1)Imhotep Virtual Medical School
 
Reproductive System Pathology_FM Breast and FM Reproductive Systems
Reproductive System Pathology_FM Breast and FM Reproductive SystemsReproductive System Pathology_FM Breast and FM Reproductive Systems
Reproductive System Pathology_FM Breast and FM Reproductive SystemsImhotep Virtual Medical School
 
Reproductive System Pathology_Male Reproductive Systems
Reproductive System Pathology_Male Reproductive SystemsReproductive System Pathology_Male Reproductive Systems
Reproductive System Pathology_Male Reproductive SystemsImhotep Virtual Medical School
 
Nervous System Pathology_A Case-based Learning Approach
Nervous System Pathology_A Case-based Learning ApproachNervous System Pathology_A Case-based Learning Approach
Nervous System Pathology_A Case-based Learning ApproachImhotep Virtual Medical School
 
CVS Function, Regulation of the Heart and Overview of Therapeutic Goals in CV...
CVS Function, Regulation of the Heart and Overview of Therapeutic Goals in CV...CVS Function, Regulation of the Heart and Overview of Therapeutic Goals in CV...
CVS Function, Regulation of the Heart and Overview of Therapeutic Goals in CV...Imhotep Virtual Medical School
 
Cardiovascular Pathology Case-based_Gross and Microscopic
Cardiovascular Pathology Case-based_Gross and MicroscopicCardiovascular Pathology Case-based_Gross and Microscopic
Cardiovascular Pathology Case-based_Gross and MicroscopicImhotep Virtual Medical School
 
Clinical Pharmacology for Medical Students_USMLE Step 1 & 2 Review
Clinical Pharmacology for Medical Students_USMLE Step 1 & 2 ReviewClinical Pharmacology for Medical Students_USMLE Step 1 & 2 Review
Clinical Pharmacology for Medical Students_USMLE Step 1 & 2 ReviewImhotep Virtual Medical School
 
Make the Dx_ A Case-based Intro to Select Cardiovascular and Respiratory Dise...
Make the Dx_ A Case-based Intro to Select Cardiovascular and Respiratory Dise...Make the Dx_ A Case-based Intro to Select Cardiovascular and Respiratory Dise...
Make the Dx_ A Case-based Intro to Select Cardiovascular and Respiratory Dise...Imhotep Virtual Medical School
 
Myocardial infarction_ Causes, Symptoms, Diagnosis, Treatment, and Pathology
Myocardial infarction_ Causes, Symptoms, Diagnosis, Treatment, and PathologyMyocardial infarction_ Causes, Symptoms, Diagnosis, Treatment, and Pathology
Myocardial infarction_ Causes, Symptoms, Diagnosis, Treatment, and PathologyImhotep Virtual Medical School
 
Autonomic Nervous System Physiology and Pharmacology_Overview| Review of ANS
Autonomic Nervous System Physiology and Pharmacology_Overview| Review of ANSAutonomic Nervous System Physiology and Pharmacology_Overview| Review of ANS
Autonomic Nervous System Physiology and Pharmacology_Overview| Review of ANSImhotep Virtual Medical School
 

Plus de Imhotep Virtual Medical School (20)

Oncologic Pathology_A Case-based Organ Systems Review (USMLE Step 1)
Oncologic Pathology_A Case-based Organ Systems Review (USMLE Step 1)Oncologic Pathology_A Case-based Organ Systems Review (USMLE Step 1)
Oncologic Pathology_A Case-based Organ Systems Review (USMLE Step 1)
 
Pathology and Pathophysiology of Shock
Pathology and Pathophysiology of ShockPathology and Pathophysiology of Shock
Pathology and Pathophysiology of Shock
 
Drugs Used In Disorders of the Reproductive System
Drugs Used In Disorders of the Reproductive SystemDrugs Used In Disorders of the Reproductive System
Drugs Used In Disorders of the Reproductive System
 
Reproductive System Pathology_FM Breast and FM Reproductive Systems
Reproductive System Pathology_FM Breast and FM Reproductive SystemsReproductive System Pathology_FM Breast and FM Reproductive Systems
Reproductive System Pathology_FM Breast and FM Reproductive Systems
 
Reproductive System Pathology_Male Reproductive Systems
Reproductive System Pathology_Male Reproductive SystemsReproductive System Pathology_Male Reproductive Systems
Reproductive System Pathology_Male Reproductive Systems
 
Nervous System Pathology_A Case-based Learning Approach
Nervous System Pathology_A Case-based Learning ApproachNervous System Pathology_A Case-based Learning Approach
Nervous System Pathology_A Case-based Learning Approach
 
CVS Function, Regulation of the Heart and Overview of Therapeutic Goals in CV...
CVS Function, Regulation of the Heart and Overview of Therapeutic Goals in CV...CVS Function, Regulation of the Heart and Overview of Therapeutic Goals in CV...
CVS Function, Regulation of the Heart and Overview of Therapeutic Goals in CV...
 
Cardiovascular Pathology Case-based_Gross and Microscopic
Cardiovascular Pathology Case-based_Gross and MicroscopicCardiovascular Pathology Case-based_Gross and Microscopic
Cardiovascular Pathology Case-based_Gross and Microscopic
 
HIV / AIDS Pathology
HIV / AIDS PathologyHIV / AIDS Pathology
HIV / AIDS Pathology
 
Sepsis & Septic Shock
Sepsis & Septic ShockSepsis & Septic Shock
Sepsis & Septic Shock
 
Drugs Used in infectious Disease_Antibiotics
Drugs Used in infectious Disease_AntibioticsDrugs Used in infectious Disease_Antibiotics
Drugs Used in infectious Disease_Antibiotics
 
Hematopoietic and Lymphoid Systems Pathology
Hematopoietic and Lymphoid Systems  PathologyHematopoietic and Lymphoid Systems  Pathology
Hematopoietic and Lymphoid Systems Pathology
 
Drugs Used in Neoplastic Disorders
Drugs Used in Neoplastic DisordersDrugs Used in Neoplastic Disorders
Drugs Used in Neoplastic Disorders
 
Neoplasia & Oncologic Pathology
Neoplasia & Oncologic PathologyNeoplasia & Oncologic Pathology
Neoplasia & Oncologic Pathology
 
Clinical Pharmacology for Medical Students_USMLE Step 1 & 2 Review
Clinical Pharmacology for Medical Students_USMLE Step 1 & 2 ReviewClinical Pharmacology for Medical Students_USMLE Step 1 & 2 Review
Clinical Pharmacology for Medical Students_USMLE Step 1 & 2 Review
 
Make the Dx_ A Case-based Intro to Select Cardiovascular and Respiratory Dise...
Make the Dx_ A Case-based Intro to Select Cardiovascular and Respiratory Dise...Make the Dx_ A Case-based Intro to Select Cardiovascular and Respiratory Dise...
Make the Dx_ A Case-based Intro to Select Cardiovascular and Respiratory Dise...
 
Myocardial infarction_ Causes, Symptoms, Diagnosis, Treatment, and Pathology
Myocardial infarction_ Causes, Symptoms, Diagnosis, Treatment, and PathologyMyocardial infarction_ Causes, Symptoms, Diagnosis, Treatment, and Pathology
Myocardial infarction_ Causes, Symptoms, Diagnosis, Treatment, and Pathology
 
Basic CXR Interpretation_Diagnostic Radiographs
Basic CXR Interpretation_Diagnostic RadiographsBasic CXR Interpretation_Diagnostic Radiographs
Basic CXR Interpretation_Diagnostic Radiographs
 
Electrocardiogram (ECG) Interpretation_Module 1 of 2
Electrocardiogram (ECG) Interpretation_Module 1 of 2Electrocardiogram (ECG) Interpretation_Module 1 of 2
Electrocardiogram (ECG) Interpretation_Module 1 of 2
 
Autonomic Nervous System Physiology and Pharmacology_Overview| Review of ANS
Autonomic Nervous System Physiology and Pharmacology_Overview| Review of ANSAutonomic Nervous System Physiology and Pharmacology_Overview| Review of ANS
Autonomic Nervous System Physiology and Pharmacology_Overview| Review of ANS
 

Dernier

Comparative Literature in India by Amiya dev.pptx
Comparative Literature in India by Amiya dev.pptxComparative Literature in India by Amiya dev.pptx
Comparative Literature in India by Amiya dev.pptxAvaniJani1
 
6 ways Samsung’s Interactive Display powered by Android changes the classroom
6 ways Samsung’s Interactive Display powered by Android changes the classroom6 ways Samsung’s Interactive Display powered by Android changes the classroom
6 ways Samsung’s Interactive Display powered by Android changes the classroomSamsung Business USA
 
4.9.24 School Desegregation in Boston.pptx
4.9.24 School Desegregation in Boston.pptx4.9.24 School Desegregation in Boston.pptx
4.9.24 School Desegregation in Boston.pptxmary850239
 
ClimART Action | eTwinning Project
ClimART Action    |    eTwinning ProjectClimART Action    |    eTwinning Project
ClimART Action | eTwinning Projectjordimapav
 
4.9.24 Social Capital and Social Exclusion.pptx
4.9.24 Social Capital and Social Exclusion.pptx4.9.24 Social Capital and Social Exclusion.pptx
4.9.24 Social Capital and Social Exclusion.pptxmary850239
 
ARTERIAL BLOOD GAS ANALYSIS........pptx
ARTERIAL BLOOD  GAS ANALYSIS........pptxARTERIAL BLOOD  GAS ANALYSIS........pptx
ARTERIAL BLOOD GAS ANALYSIS........pptxAneriPatwari
 
Scientific Writing :Research Discourse
Scientific  Writing :Research  DiscourseScientific  Writing :Research  Discourse
Scientific Writing :Research DiscourseAnita GoswamiGiri
 
Employablity presentation and Future Career Plan.pptx
Employablity presentation and Future Career Plan.pptxEmployablity presentation and Future Career Plan.pptx
Employablity presentation and Future Career Plan.pptxryandux83rd
 
Transaction Management in Database Management System
Transaction Management in Database Management SystemTransaction Management in Database Management System
Transaction Management in Database Management SystemChristalin Nelson
 
Healthy Minds, Flourishing Lives: A Philosophical Approach to Mental Health a...
Healthy Minds, Flourishing Lives: A Philosophical Approach to Mental Health a...Healthy Minds, Flourishing Lives: A Philosophical Approach to Mental Health a...
Healthy Minds, Flourishing Lives: A Philosophical Approach to Mental Health a...Osopher
 
An Overview of the Calendar App in Odoo 17 ERP
An Overview of the Calendar App in Odoo 17 ERPAn Overview of the Calendar App in Odoo 17 ERP
An Overview of the Calendar App in Odoo 17 ERPCeline George
 
Oppenheimer Film Discussion for Philosophy and Film
Oppenheimer Film Discussion for Philosophy and FilmOppenheimer Film Discussion for Philosophy and Film
Oppenheimer Film Discussion for Philosophy and FilmStan Meyer
 
Beauty Amidst the Bytes_ Unearthing Unexpected Advantages of the Digital Wast...
Beauty Amidst the Bytes_ Unearthing Unexpected Advantages of the Digital Wast...Beauty Amidst the Bytes_ Unearthing Unexpected Advantages of the Digital Wast...
Beauty Amidst the Bytes_ Unearthing Unexpected Advantages of the Digital Wast...DhatriParmar
 
Tree View Decoration Attribute in the Odoo 17
Tree View Decoration Attribute in the Odoo 17Tree View Decoration Attribute in the Odoo 17
Tree View Decoration Attribute in the Odoo 17Celine George
 
Grade Three -ELLNA-REVIEWER-ENGLISH.pptx
Grade Three -ELLNA-REVIEWER-ENGLISH.pptxGrade Three -ELLNA-REVIEWER-ENGLISH.pptx
Grade Three -ELLNA-REVIEWER-ENGLISH.pptxkarenfajardo43
 

Dernier (20)

Faculty Profile prashantha K EEE dept Sri Sairam college of Engineering
Faculty Profile prashantha K EEE dept Sri Sairam college of EngineeringFaculty Profile prashantha K EEE dept Sri Sairam college of Engineering
Faculty Profile prashantha K EEE dept Sri Sairam college of Engineering
 
Comparative Literature in India by Amiya dev.pptx
Comparative Literature in India by Amiya dev.pptxComparative Literature in India by Amiya dev.pptx
Comparative Literature in India by Amiya dev.pptx
 
6 ways Samsung’s Interactive Display powered by Android changes the classroom
6 ways Samsung’s Interactive Display powered by Android changes the classroom6 ways Samsung’s Interactive Display powered by Android changes the classroom
6 ways Samsung’s Interactive Display powered by Android changes the classroom
 
4.9.24 School Desegregation in Boston.pptx
4.9.24 School Desegregation in Boston.pptx4.9.24 School Desegregation in Boston.pptx
4.9.24 School Desegregation in Boston.pptx
 
ClimART Action | eTwinning Project
ClimART Action    |    eTwinning ProjectClimART Action    |    eTwinning Project
ClimART Action | eTwinning Project
 
4.9.24 Social Capital and Social Exclusion.pptx
4.9.24 Social Capital and Social Exclusion.pptx4.9.24 Social Capital and Social Exclusion.pptx
4.9.24 Social Capital and Social Exclusion.pptx
 
ARTERIAL BLOOD GAS ANALYSIS........pptx
ARTERIAL BLOOD  GAS ANALYSIS........pptxARTERIAL BLOOD  GAS ANALYSIS........pptx
ARTERIAL BLOOD GAS ANALYSIS........pptx
 
Scientific Writing :Research Discourse
Scientific  Writing :Research  DiscourseScientific  Writing :Research  Discourse
Scientific Writing :Research Discourse
 
Employablity presentation and Future Career Plan.pptx
Employablity presentation and Future Career Plan.pptxEmployablity presentation and Future Career Plan.pptx
Employablity presentation and Future Career Plan.pptx
 
Transaction Management in Database Management System
Transaction Management in Database Management SystemTransaction Management in Database Management System
Transaction Management in Database Management System
 
prashanth updated resume 2024 for Teaching Profession
prashanth updated resume 2024 for Teaching Professionprashanth updated resume 2024 for Teaching Profession
prashanth updated resume 2024 for Teaching Profession
 
Healthy Minds, Flourishing Lives: A Philosophical Approach to Mental Health a...
Healthy Minds, Flourishing Lives: A Philosophical Approach to Mental Health a...Healthy Minds, Flourishing Lives: A Philosophical Approach to Mental Health a...
Healthy Minds, Flourishing Lives: A Philosophical Approach to Mental Health a...
 
An Overview of the Calendar App in Odoo 17 ERP
An Overview of the Calendar App in Odoo 17 ERPAn Overview of the Calendar App in Odoo 17 ERP
An Overview of the Calendar App in Odoo 17 ERP
 
Introduction to Research ,Need for research, Need for design of Experiments, ...
Introduction to Research ,Need for research, Need for design of Experiments, ...Introduction to Research ,Need for research, Need for design of Experiments, ...
Introduction to Research ,Need for research, Need for design of Experiments, ...
 
Oppenheimer Film Discussion for Philosophy and Film
Oppenheimer Film Discussion for Philosophy and FilmOppenheimer Film Discussion for Philosophy and Film
Oppenheimer Film Discussion for Philosophy and Film
 
Beauty Amidst the Bytes_ Unearthing Unexpected Advantages of the Digital Wast...
Beauty Amidst the Bytes_ Unearthing Unexpected Advantages of the Digital Wast...Beauty Amidst the Bytes_ Unearthing Unexpected Advantages of the Digital Wast...
Beauty Amidst the Bytes_ Unearthing Unexpected Advantages of the Digital Wast...
 
Tree View Decoration Attribute in the Odoo 17
Tree View Decoration Attribute in the Odoo 17Tree View Decoration Attribute in the Odoo 17
Tree View Decoration Attribute in the Odoo 17
 
Mattingly "AI & Prompt Design" - Introduction to Machine Learning"
Mattingly "AI & Prompt Design" - Introduction to Machine Learning"Mattingly "AI & Prompt Design" - Introduction to Machine Learning"
Mattingly "AI & Prompt Design" - Introduction to Machine Learning"
 
Mattingly "AI & Prompt Design: Large Language Models"
Mattingly "AI & Prompt Design: Large Language Models"Mattingly "AI & Prompt Design: Large Language Models"
Mattingly "AI & Prompt Design: Large Language Models"
 
Grade Three -ELLNA-REVIEWER-ENGLISH.pptx
Grade Three -ELLNA-REVIEWER-ENGLISH.pptxGrade Three -ELLNA-REVIEWER-ENGLISH.pptx
Grade Three -ELLNA-REVIEWER-ENGLISH.pptx
 

General Princples of Pharmacology_Approach to Learning Pharmacology

  • 1. GENERAL PRINCIPLES OF PHARMACOLOGY Approach to Learning Pharmacology Marc Imhotep Cray, M.D.
  • 2. Marc Imhotep Cray, M.D. Pedagogical Approach 2  This course sets out to provide an understanding of scientific and clinical pharmacology within the framework of biochemistry, physiology and pathophysiology, travelling from drug effects on molecular targets to their effects on the whole-organism. In other words,  You will be learning pharmacology using an approach that integrates the actions of medications (drugs) starting from the level of molecular targets (biologic effects) to the level of the clinical patient (therapeutic and adverse effects), relying on kindred basic medical sciences as the scaffolding.
  • 3. Marc Imhotep Cray, M.D. “What Is Pharmacology?” …“Medical pharmacology is a bridge between basic medical science and clinical medicine. It makes use of all the disciplines that comprise the scientific foundation of clinical medicine; including anatomy, physiology, pathophysiology, pathology and immunology biochemistry, molecular and cell biology, epidemiology, genetics and genomics. Hence…it is particularly useful for pre-clinical student to view and engage the subject as a major horizontal and vertical integrator, as it pulls together all the different strands of the basic medical science years and simultaneously introduces one to the cornerstone of modern clinical therapeutics, i.e. drugs”… Cray MI. Integrated Scientific and Clinical Pharmacology: A Course Syllabus and Digital Guidebook for Medical Students. Atlanta, Georgia: IVMS, 2015; 4. 3
  • 4. 4 Psychology Clinical Medicine & Therapeutics Veterinary Medicine Pharmacy Biotechnology Pathology Chemistry Psycho- pharmacology Clinical Pharmacology Veterinary Pharmacology Pharmaceutical Science Bio- pharmaceutics Toxicology Medicinal Chemistry PHARMACOLOGY Pharmacokinetics & Pharmacodynamics Biochemical pharmacology Molecular pharmacology Chemotherapy Systems pharmacology Neuropharmacology Respiratory pharmacology Gastrointestinal pharmacology Immunopharmacology Cardiovascular pharmacology Endocrine pharmacology Pharmacogenetics Pharmacogenomics Pharmacoepidemiology Pharmacoeconomics Genetics Genomics Clinical Epidemiology Health Economics The Scope of Pharmacology
  • 5. Marc Imhotep Cray, M.D. Importance of the GPs of Pharmacology 5  The general principles of pharmacology are absolutely key to gaining an understanding of how drugs exert their therapeutic and adverse effects  General principles are at the core of organ-systems pharmacology, as you will be applying these principles during the study of drug classes/organ-systems pharmacology  Therefore, a strong knowledge of general principles will help you most with your study of organ-systems pharmacology, as well as the prudent application of pharmacotherapeutics during your clinical medicine training
  • 6. Marc Imhotep Cray, M.D. Learning Objectives 6 1. To understand some key terms and concepts related to the general principles of pharmacology (pharmacokinetics and pharmacodynamics). 2. To understand the approach to the study of disease within a medical pharmacology and therapeutics framework. 3. To understand how one should approach the study of pharmacology and the rational underlying. (Main Objective) Note: The terms, concepts and mechanisms provided herein are not intended as complete discussions. Rather, they are only introduced here as “stair-steps” and to encapsulate Objective 3. They will be explored in more detail further in the course of study.
  • 7. Marc Imhotep Cray, M.D. Lecture Outline 7  Approach to Learning Pharmacology o Key Terms and Concepts o Mechanisms of Drug Action • 4 Levels of complexity  Approach to Disease  Approach to Reading & Studying Pharmacology o “The seven key questions”  Key Points Capsule  Comprehension Q & A  Further Study: IVMS Tools and Resources
  • 8. Marc Imhotep Cray, M.D. 1. Approach to Learning Pharmacology 8 Pharmacology is best learned by a systematic approach  understanding physiology of body  recognizing every medication (drug) has desirable and undesirable effects and  being aware biochemical and pharmacologic properties of a drug affects its characteristics such as… o duration of action o volume of distribution o passage through blood-brain barrier o mechanism of elimination, and o route of administration
  • 9. Marc Imhotep Cray, M.D. Approach to Learning Pharmacology (2) 9  Rather than memorizing characteristics of a medication, one should strive to learn underlying rationale for example  Second-generation antihistamine agents are less lipid soluble than first-generation antihistamines  therefore 2nd Gen. agents do not cross blood-brain barrier (BBB) as readily thus, 2nd Gen. antihistamines are not as sedating  b/c they both bind histamine H1 receptor, efficacy is same in treating conditions for which they are indicated (e.g., allergic rhinitis )
  • 10. Marc Imhotep Cray, M.D. Key Terms and Concepts 10  Pharmacology: The study of substances that interact with living systems through biochemical processes.  Drug (medication): A substance used in prevention, diagnosis, or treatment of a disease or prevention of reproduction.  Toxicology: A branch of pharmacology that studies undesirable effects of chemicals on living organisms.  Food and Drug Administration (FDA): Federal agency responsible for safety & efficacy of all drugs in U.S, as well as food & cosmetics.  Adverse effect: Also known as side effect; all unintended actions of a drug that result from lack of specificity of drug action. N.B. All drugs are capable of producing adverse effects>>>toxicity
  • 11. Marc Imhotep Cray, M.D. Key Terms and Concepts(2) 11 Classically there are two major divisions of pharmacology: pharmacodynamics and pharmacokinetics  Pharmacodynamics (PD): The actions of a drug on a living organism, including mechanisms of action and receptor interaction.  How the drug affects the body  Pharmacokinetics (PK): The actions of the living organism on the drug, including absorption, distribution, and elimination.  How the body affects the drug Third emerging division  Pharmacogenomics: study of how genomic makeup affects PD &PK  affects drug selection and application to individual patients based on interindividual variations in the handling of drugs based on genetics
  • 12. Marc Imhotep Cray, M.D. Pharmacogenomics 12 “Pharmacogenomics may hold the opportunity of allowing practitioners to integrate a molecular understanding of the basis of disease with an individual's genomic makeup to prescribe personalized, highly effective, and safe therapies.” Roden DM. (2012) Ch. 5 Principles of Clinical Pharmacology. In: Longo DL, Fauci AS, et al. Harrison's Principles of Internal Medicine,18th Ed., McGraw-Hill 2012, 33. NB “Drug-gene testing is also called pharmacogenomics, or pharmacogenetics. All terms characterize the study of how your genes affect your body's response to medications.” Link to see animation and transcript: http://mayoresearch.mayo.edu/center- for-individualized-medicine/drug-gene-testing.asp
  • 13. Marc Imhotep Cray, M.D. Major PD and PK Components & Parameters 13 Trevor AJ, Katzung BG, Kruidering-Hall M , Masters SB. Katzung & Trevor's Pharmacology Examination & Board Review 10th Ed. New York: McGraw-Hill, 2013.
  • 14. Marc Imhotep Cray, M.D. Relationship between PK and PD Pharmacokinetics Pharmacodynamics Dose of drug Drug concentration in target organ over time Mechanism and magnitude of drug effect Absorption Distribution Biotransformation Excretion Receptor binding Signal transduction Physiological effect Redrawn after Brenner GM and Stevens CW. Pharmacology 4th ed. Sanders, 2014. 14
  • 15. 15 Key Terms and Concepts(4)  Potency of drug (x-axis on dose-response curve): Relative amount of drug needed to produce a given response  determined by amt. of drug that reaches site of action and by affinity of drug for receptor  Efficacy (y-axis on dose-response curve): Drug effect as maximum response it is able to produce  determined by number of drug-receptor complexes and ability of receptor to be activated once bound (intrinsic activity) • EC50 [also ED50 in many text] refers to drug concentration that produces 50 percent of maximal response (Graded curve); whereas • ED50 (Quantal curve) refers to drug dose that is pharmacologically effective in 50 percent of population NB-There are two types of dose-response curves--graded and quantal-- each provides useful information for therapeutic decisions…see next slide
  • 16. Marc Imhotep Cray, M.D. Dose-Response (Concentration) Curves 16 Bardal KS, Waechter JE, Martin DS. Applied Pharmacology. St. Louis: Saunders, 2011. Graded dose-response curve Quantal dose-response curve: A graph of the fraction of a population that shows a specified response at progressively increasing doses A graph of increasing response to increasing drug concentration or dose
  • 17. Marc Imhotep Cray, M.D. Key Terms and Concepts(5) 17  Agonist: A drug that activates its receptor upon binding  Affinity and intrinsic activity; whereas, antagonists have affinity but no IA Antagonist: A drug that attenuates effect of an agonist  Can be competitive or non-competitive each of which can be reversible or irreversible  Pharmacologic antagonist: A drug that binds without activating its receptor and thereby prevents activation by an agonist  Competitive antagonist: A pharmacologic antagonist that can be overcome by increasing conc. of agonist  Non-competitive antagonist: binds to an allosteric (non-agonist) site on receptor to prevent activation of receptor
  • 18. Marc Imhotep Cray, M.D. Key Terms and Concepts(6) 18  Reversible antagonist: binds non-covalently to receptor therefore can be “washed out”  A pharmacologic antagonist that can be overcome by increasing agonist conc.  Irreversible antagonist: binds covalently to receptor and cannot be displaced by either competing ligands or washing  A pharmacologic antagonist that cannot be overcome by increasing agonist conc.  Physiologic antagonist: A drug that counters effects of another by binding to a different receptor and causing opposing effects
  • 19. 19 Potential mechanisms of drug interaction with a receptor Katzung BG, editor: Basic & Clinical Pharmacology, 12th ed. New York: McGraw-Hill, 2012; Fig. 1–3.) Possible effects resulting from these interactions are diagrammed in dose- response curves at right  Ligand: A substance that forms a complex with a biomolecule (receptor) to serve a biological purpose  Receptor: A molecule to which a ligand (drug) binds to bring about a change in function of biologic system  Receptor site: Specific region of receptor molecule to which drug binds  Effector: Component of a system that accomplishes biologic effect after receptor is activated by an agonist  often a channel or enzyme molecule may be part of receptor molecule itself
  • 20. Marc Imhotep Cray, M.D. 20 Routes of administration (RoA) Path by which a drug, fluid or poison is taken into body  Routes can also be classified based on where target of action is  Action may be topical (local)  Enteral (system-wide effect, but delivered through GIT), or  Parenteral (systemic action, but delivered by routes other than GI tract) Key Terms and Concepts(7)
  • 21. 21 Key Terms and Concepts(8)  Route of administration: Drug may be delivered  intravenously (IV) for delivery directly into bloodstream  intramuscularly (IM), and  subcutaneously (SC)  Medication may be  depot and slow release  inhalant for rapid absorption & delivery to bronchi & lungs  sublingual to bypass first-pass effect  intrathecal for agents that penetrate BBB poorly  rectal to avoid hepatic first-pass effect and for N/V, and  topical administration when local effect is desired such as dermatologic or ophthalmic agents Lippincott Illustrated Reviews: Pharmacology Sixth Ed. Wolters Kluwer, 2015.
  • 22. Marc Imhotep Cray, M.D. Key Terms and Concepts(9) 22 Absorption: Movement of a drug from administration site into blood stream usually requiring crossing of one or more biologic membranes Important parameters include • lipid solubility • ionization • size of molecule and • presence of a transport mechanism
  • 23. Marc Imhotep Cray, M.D. Key Terms and Concepts(10) 23  Bioavailability: The percentage of an ingested drug that is actually absorbed into bloodstream  By definition, intravascular doses have 100% bioavailability, f = 1. Lippincott Illustrated Reviews: Pharmacology Sixth Ed. Wolters Kluwer, 2015.  Factors that influence bioavailability: o First-pass hepatic metabolism o Solubility of the drug o Chemical instability o Nature of the drug formulation
  • 24. Marc Imhotep Cray, M.D. Key Terms and Concepts(11) 24  Volume of distribution (Vd): The size of “compartment” into which a drug is distributed following absorption  ratio of amt. of drug in body to drug conc. in plasma or blood Units=liters  determined by equation: Vd = Dose (mg) drug administered/Initial plasma concentration (mg/L)
  • 25. Marc Imhotep Cray, M.D. Key Terms and Concepts(12) 25  Elimination: Process by which a drug is removed from body, generally by either metabolism (biotransformation) or excretion  Elimination=biotransformation (liver…) + excretion (kidneys…)  Elimination follows various kinetic models For example  First-order kinetics describes most circumstances means that rate of drug elimination depends on concentration of drug in plasma as described by equation: Rate of elimination from body = Constant × Drug concentration  Zero-order kinetics: It is less common (PEA) means that rate of elimination is constant does not depend on the plasma drug concentration • consequence of a circumstance such as saturation of liver enzymes or saturation of kidney transport mechanisms
  • 26. Marc Imhotep Cray, M.D. Key Terms and Concepts(13) 26  Clearance: Ratio of rate of elimination of a drug to concentration of drug in plasma or blood  Units: volume/time, eg, mL/min or L/h  Half-life: Time required for amount of drug in body or blood to fall by 50%  For drugs eliminated by first-order kinetics this number is a constant regardless of concentration  Units: time
  • 27. Marc Imhotep Cray, M.D. Mechanisms of Drug Action Drug effects are produced by altering normal functions of cells and tissues via one of four general mechanisms: 1. Interaction with receptors (major) Ligand-activated ion channels G-protein–coupled receptors  Gαs-coupled receptors  Gαi (Ginhibitory)-coupled receptors  Gq (and G11)-coupled receptors Intracellular nuclear receptors Receptor-activated tyrosine kinases 2. Nonspecific chemical or physical interactions (least common)  e.g., antacids 3. Antimetabolite action  e.g., ChemoTx agents 4. Alteration of the activity of enzymes  increasing or decreasing 27 Important Notes: • Drugs do not produce new function/s in body, but rather augment nml physiologic and biochemical mechanisms • No drug has a single action, but rather, both therapeutic & adverse actions or multiple effects • Drug vs poison is dose related, as all drugs are poisons when introduced at a high enough dose
  • 28. Marc Imhotep Cray, M.D. 28 Drugs act at four different levels: 1) Molecular: protein molecules are immediate targets for most drugs. Action here translates into actions at next level 2) Cellular: biochemical and other components of cells participate in the process of transduction 3) Tissue: the function of heart, skin, lungs, etc., is then altered 4) System: the function of the cardiovascular, nervous, gastrointestinal system, etc., is then altered Mechanisms of Drug Action (2)
  • 29. Marc Imhotep Cray, M.D. Mechanisms of Drug Action (3) 29  To most clearly understand pharmacologic actions of drugs (agonist and antagonist) it is necessary to know:  which molecular targets are affected by the drug,  nature of this molecular interaction,  nature of the transduction system (the cellular response),  types of tissue that express the molecular target and  mechanisms by which the tissue influences the body system NB. It is important to consider MOA of drugs at each of the four levels of complexity.
  • 30. Marc Imhotep Cray, M.D. The four levels of MOA illustrated 30 Propranolol, a β adrenergic antagonist used to treat several diseases including angina pectoris, a cardiac condition resulting from localized ischemia (i.e. insufficient blood flow) in heart:  At the molecular level, propranolol is a competitive and reversible antagonist to action of epinephrine (Epi) and norepinephrine (NE) on cardiac β adrenoceptors
  • 31. Marc Imhotep Cray, M.D. Four levels of MOA cont. 31  At the cellular level, propranolol prevents β adrenergic agonism from elevating intracellular cyclic adenosine monophosphate (cAMP), initiating protein phosphorylation, Ca2+ mobilization and oxidative metabolism  At the tissue level, propranolol prevents β adrenergic agonism from increasing contractile force of heart and heart rate, i.e. it has negative inotropic and negative chronotropic effects
  • 32. Marc Imhotep Cray, M.D. Four levels of MOA cont. 32  At a system level, propranolol improves cardiovascular function  It reduces heart's β adrenergic responses to sympathetic nervous system activity thereby decreasing requirements for blood flow (O2 demand) in heart tissue useful if blood supply is limited (e.g. in coronary artery disease)
  • 33. Marc Imhotep Cray, M.D. 2. Approach to Disease 33 Physicians approach clinical situations by  taking a history (asking questions)  performing a physical examination  obtaining selective laboratory and imaging tests, and  then formulating a diagnosis The synthesis of history, physical examination, and imaging or laboratory tests is called the clinical database After reaching a diagnosis a treatment plan is initiated, and patient is followed for a clinical response
  • 34. Marc Imhotep Cray, M.D. Approach to Disease (2) 34  Rational understanding of disease (pathologic, microbiologic, immunologic and (or) behavioral) and plans for treatment (therapeutics) are best acquired by learning about normal human processes on a basic science level (physiology, biochemistry neuro and behavioral science)  likewise, being aware of how disease alters normal physiologic processes is also best understood on a basic science level (pathology, pathogenesis & pathophysiology)  Sn & Sx of disease
  • 35. Marc Imhotep Cray, M.D. Approach to Disease (3) 35  Pharmacology and therapeutics require also ability to tailor correct medication (drug) to patient’s situation and awareness of medication’s adverse effect profile  Sometimes, a patient has an adverse reaction to a drug as chief complaint one must be able to identify medication as culprit  Again, an understanding of underlying basic science allows for more rational analysis and drug (medication) choices
  • 36. Marc Imhotep Cray, M.D. 3. Approach to Reading & Studying Pharmacology 36  There are seven key questions that help to stimulate application of basic science information to clinical setting These are: 1. Which medications is most likely to achieve desired therapeutic effect and/or is responsible for described symptoms or signs? 2. What is likely mechanism for clinical effect(s) and adverse effect(s) of medication? 3. What is basic pharmacologic profile (e.g., absorption, elimination) for medications in a certain class, and what are differences among the agents within the class?
  • 37. Marc Imhotep Cray, M.D. Approach to Reading and Studying Pharm (2) 37 4. Given basic pharmacologic definitions such as therapeutic index (TI) or certain safety factor (TD1/ED99), or median lethal dose (LD50), how do medications compare in their safety profile? 5. Given a particular clinical situation with described unique patient characteristics, which medication is most appropriate? 6. What is best treatment toxic effect of a medication? 7. What are drug-drug interactions to be cautious about regarding a particular medication? In the following slides we will discuss a bit more about each of these seven key questions, explaining how they help to stimulate application of basic science information to the clinical setting.
  • 38. Marc Imhotep Cray, M.D. 1. Which drug is most likely responsible for described symptoms or signs? 38  One must be aware of various effects, both desirable (therapeutic) and undesirable (ADR), produced by particular medications  Knowledge of desirable therapeutic effects is essential in selecting appropriate drug for particular clinical application  Likewise, an awareness of its adverse effects (AE) is necessary, b/c patients may present with a complaint caused by a drug effect unaware that their symptoms are b/c of the prescribed drug  only by being aware of common and dangerous effects can one arrive at correct diagnosis  Learners are encouraged not to merely memorize comparative adverse effect profiles of drugs but rather to understand underlying biochemical and physiological mechanisms
  • 39. Marc Imhotep Cray, M.D. 2. What is mechanism for clinical effect(s) and adverse effect(s) of drug? 39 As noted one should strive to learn underlying physiologic, biochemical, and (or) cellular explanation for drug effect  allows for rational choice of an alternative agent or reasonable choice of an agent to alleviate symptoms or explanatory advice to pt. regarding behavioral changes to diminish any adverse effects o For example, if a 60-year-old woman who takes medications for osteoporosis complains of severe “heartburn” one may be suspicious, knowing that bisphosphonate medication alendronate can cause esophagitis • Instruction to patient to take medication while sitting upright and remaining upright for at least 30 minutes would be proper course of action, b/c gravity will assist in keeping alendronate in stomach rather than allowing regurgitation into distal esophagus
  • 40. Marc Imhotep Cray, M.D. 3. What is basic pharmacologic profile for drugs in a certain class, and what are differences among agents within said class? 40  Understanding pharmacologic profile of medications allows for rational therapeutics   instead of memorizing separate profiles for every medication, grouping drugs together into classes allows for more efficient learning and better comprehension  Excellent starting point is to study how a prototype drug within a drug class organized by structure or mechanism of action may be used to treat a condition (such as hypertension)  o Then within each category of agents, one should try to identify important subclasses or drug differences
  • 41. Marc Imhotep Cray, M.D. Pharmacologic profile of a drug and differences among agents within class (2) 41  For example, anti-hypertensive agents can be categorized as  diuretic agents  β-adrenergic-blocking agents  calcium-channel-blocking agents, and  renin-angiotensin system inhibitors= ACEI & ARBs  Within subclassification of renin-angiotensin system inhibitors, angiotensin converting enzyme inhibitors (ACEI) can cause adverse effect of a dry cough caused by increase in bradykinin brought about by enzyme blockade  instead, angiotensin-1 receptor blockers (ARBs) do not affect bradykinin levels and so do not cause cough as often
  • 42. Marc Imhotep Cray, M.D. 4. Given basic pharmacologic definitions such as (TI) or (TD1/ED99), or (LD50), how do medications compare in their safety profile? 42  Therapeutic index (TI): Defined as TD50/ED50 (ratio of dose that produces a toxic effect in half population to dose that produces desired effect in half population)  Certain safety factor (TD1/ED99): Defined as ratio of dose that produces toxic effect in 1 percent of population to dose that produces desired effect in 99 percent of population also known as standard safety measure  Median lethal dose (LD50): Defined as the median lethal dose, the dose that will kill half the population
  • 43. Marc Imhotep Cray, M.D. TI or TD1/ED99, or LD50 (2) 43  Based on these definitions, a desirable medication would have a high therapeutic index (toxic dose is many times that of efficacious dose), high certain safety factor, and high median lethal dose (much higher than therapeutic dose)  Likewise, medications such as digoxin that have a low therapeutic index require careful monitoring of bld levels and vigilance for adverse effects
  • 44. Marc Imhotep Cray, M.D. 5. Given a particular clinical situation with described unique patient characteristics, which medication is most appropriate? 44 One must weigh various advantages and disadvantages, as well as different patient attributes Some of those may include  compliance with medications  allergies to medications  liver or renal insufficiency  age  coexisting medical disorders, and  other medications
  • 45. Marc Imhotep Cray, M.D. Unique patient characteristics (2) 45 One must be able to sift through medication profile and identify most dangerous adverse effects For example,  if a patient is already taking a monoamine-oxidase- inhibiting agent (Selegiline) for depression then adding a serotonin reuptake inhibitor [Fluoxetine (Prozac)] would be potentially fatal, b/c serotonin syndrome may ensue (hyperthermia, muscle rigidity, death)
  • 46. 46 Unique patient characteristics (3) Patient profile Age Weight Sex Race Allergies Smoking history Alcohol history Diseases Pregnant/lactating Current therapy Intelligence Drug profile Name (generic) Class Action Pharmacokinetics Indications Contraindications/ precautions Interactions Adverse effects Dosing regimen Monitoring Overdose/Antidote Patient profile  The patient is a unique individual, with many distinguishing features that need to be taken into account during prescribing Drug profile  The drug, likewise, is unique, with its own distinguishing features  Good prescribing involves tailoring drug and dosing regimen to unique patient o Clinical pharmacology provides basis of this pharmacotherapeutic principle  Clinical pharmacology is a complex interaction betw. pt. and drug
  • 47. Marc Imhotep Cray, M.D. 6. What is best treatment for toxic effect of a medication? 47  If complications of drug therapy are present one should know proper treatment  best learned by understanding drug MOA For example, a pt. who has taken excessive opioids may develop respiratory depression, caused by either a heroin overdose or pain medication may be fatal o Tx of an opioid overdose includes • ABCs (airway, breathing, circulation) and • administration of naloxone, which is a competitive antagonist of opioids
  • 48. Marc Imhotep Cray, M.D. 7. What are the drug-drug interactions to be concerned with regarding a particular medication? 48  Patients are often prescribed multiple medications, from same practitioner or different clinicians  Pts may not be aware of drug-drug interactions thus, a clinician must compile pt. to maintain a current list of all medications (Rx, OTC, and herbal) taken by patient  Thus, one should be aware of most common and dangerous drug-drug interactions  again, understanding underlying mechanism allows for lifelong learning rather than short-term rote memorization of facts that are easily forgotten
  • 49. Marc Imhotep Cray, M.D. Drug-drug interactions (2) 49  For example, magnesium sulfate to stop preterm labor should not be used if patient is taking a calcium-channel blocking agent such as nifedipine  Magnesium sulfate acts as a competitive inhibitor of calcium and by decreasing its intracellular availability it slows down smooth muscle contraction such as in uterus  Calcium-channel blockers potentiate inhibition of calcium influx and can lead to toxic effects such as respiratory depression
  • 50. Marc Imhotep Cray, M.D. Key Points Capsule 50 ❖ Understanding the pharmacologic mechanisms of drugs allows for rational choices for therapy, fewer medication errors, and rapid recognition and reversal of toxic effects ❖ The therapeutic index, certain safety factor (TD1/ED99), and median lethal dose are various methods of describing the potential toxicity of medications ❖ There are seven key questions to stimulate the application of basic science information to the clinical arena
  • 51. Marc Imhotep Cray, M.D. Key Points Capsule (2) 51 Focus of study for each drug:  Classification and class prototype/s (as applicable)  Mechanism of action-biologic, therapeutic and adverse  Indications (therapeutic use)  Adverse effects (common vs dangerous)  Drug-drug interactions, cautions and contraindications  Pharmacokinetic properties, drug-disease interactions and other patient-specific considerations  Toxicities and antidotes (or) treatment
  • 52. Marc Imhotep Cray, M.D. “What Is Pharmacology?” A Capsule …“Medical pharmacology is a bridge between basic science and clinical medicine. It makes use of all the disciplines that comprise the scientific foundation of clinical medicine; including anatomy, physiology, pathophysiology, pathology and immunology biochemistry, molecular and cell biology, epidemiology, genetics and genomics. Hence…it is particularly useful for pre-clinical student to view and engage the subject as a major horizontal and vertical integrator, as it pulls together all the different strands of the basic medical science years and simultaneously introduces one to the cornerstone of modern clinical therapeutics, i.e. drugs”… Cray MI. Integrated Scientific and Clinical Pharmacology: A Course Syllabus and Digital Guidebook for Medical Students. Atlanta: IVMS, 2015; 4. 52
  • 54. Marc Imhotep Cray, M.D. Question 54 1. Bioavailability of an agent is maximal when the drug has which of the following qualities? A. Highly lipid soluble B. More than 100 Daltons in molecular weight C. Highly bound to plasma proteins D. Highly ionized
  • 55. Marc Imhotep Cray, M.D. Answer 55 1. A. Transport across biologic membranes and thus bioavailability is maximal with high lipid solubility.
  • 56. Marc Imhotep Cray, M.D. Question 56 2. An agent is noted to have a very low calculated volume of distribution (Vd). Which of the following is the best explanation? A. The agent is eliminated by the kidneys, and the patient has renal insufficiency. B. The agent is extensively bound to plasma proteins. C. The agent is extensively sequestered in tissue. D. The agent is eliminated by zero-order kinetics.
  • 57. Marc Imhotep Cray, M.D. Answer 57 2. B. The volume of distribution is calculated by administering a known dose of drug (mg) IV and then measuring an initial plasma concentration (mg/L). The ratio of the mass of drug given (mg) divided by the initial plasma concentration (mg/L) gives the Vd. A very low Vd may indicate extensive protein binding (drug is sequestered in the bloodstream), whereas a high Vd may indicate extensive tissue binding (drug is sequestered in the tissue).
  • 58. Marc Imhotep Cray, M.D. Question 58 3. Which of the following describes the first-pass effect? A. Inactivation of a drug as a result of the gastric acids. B. Absorption of a drug through the duodenum. C. Drug given orally is metabolized by the liver before entering the circulation. D. Drug given IV accumulates quickly in the central nervous system (CNS).
  • 59. Marc Imhotep Cray, M.D. Answer 59 3. C. The first-pass effect refers to the process in which following oral administration a drug is extensively metabolized as it initially passes through the liver, before it enters the general circulation. Liver enzymes may metabolize the agent to such an extent that the drug cannot be administered orally.
  • 60. Marc Imhotep Cray, M.D. Question 60 4. A laboratory experiment is being conducted in which a mammal is injected with a noncompetitive antagonist to the histamine receptor. Which of the following best describes this agent? A. The drug binds to the histamine receptor and partially activates it. B. The drug binds to the histamine receptor but does not activate it. C. The drug binds to the receptor, but not where histamine binds, and prevents the receptor from being activated. D. The drug irreversibly binds to the histamine receptor and renders it ineffective.
  • 61. Marc Imhotep Cray, M.D. Answer 61 4. C. A noncompetitive antagonist binds to the receptor at a site other than the agonist-binding site and renders it less effective by preventing agonist binding or preventing activation.
  • 62. Marc Imhotep Cray, M.D. Question 62 5. A 25-year-old medical student is given a prescription for asthma, which the physician states has a very high therapeutic index. Which of the statements best characterizes the drug as it relates to the therapeutic index? A. The drug’s serum levels will likely need to be carefully monitored. B. The drug is likely to cross the blood-brain barrier. C. The drug is likely to have extensive drug-drug interactions. D. The drug is unlikely to have any serious adverse effects.
  • 63. Marc Imhotep Cray, M.D. Answer 63 5. D. An agent with a high therapeutic index means the toxic dose is very much higher than the therapeutic dose, and it is less likely to produce toxic effects at therapeutic levels.
  • 64. Marc Imhotep Cray, M.D. Question 64 6. A drug M is injected IV into a laboratory subject. It is noted to have high serum protein binding. Which of the following is most likely to be increased as a result? A. Drug interaction B. Distribution of the drug to tissue sites C. Renal excretion D. Liver metabolism
  • 65. Marc Imhotep Cray, M.D. Answer 65 6. A. High protein binding means less drug to the tissue, the kidney, and the liver. Drug interaction may occur if the agent binds to the same protein site as other drugs, thus displacing drugs and increasing serum levels.
  • 66. Marc Imhotep Cray, M.D. Question 66 7. A bolus of drug K is given IV. The drug is noted to follow first-order kinetics. Which of the following describes the elimination of drug K? A. The rate of elimination of drug K is constant. B. The rate of elimination of drug K is proportional to the patient’s renal function. C. The rate of elimination of drug K is proportional to its concentration in the patient’s plasma. D. The rate of elimination of drug K is dependent on a nonlinear relationship to the plasma protein concentration.
  • 67. Marc Imhotep Cray, M.D. Answer 67 7. C. First-order kinetics means the rate of elimination of a drug is proportional to the plasma concentration.
  • 68. Marc Imhotep Cray, M.D. Further study tools and resources: 68 IVMS Online Medical Pharmacology Course (5 components): Instructor: Marc Imhotep Cray, M.D. Course Website: Link  Integrated Scientific and Clinical Pharmacology: A MS1 & MS2 Course Syllabus and Digital Guidebook (2015) • Medical Pharmacology: Core Concepts and Learning Objectives • Medical Pharmacology Case Studies • Medical Pharmacology Unit e-Notes • Medical Pharmacology Glossary of Terms e-Learning resource center: IVMS Medical Pharmacology Cloud Folder